Blog này tổng hợp các bài toán hay, các bài giảng chọn lọc về nhiều chủ đề: đại số, hình học, giải tích, số học và tổ hợp liên quan đến Toán Olympic và Toán thi ĐH.
Thứ Ba, 12 tháng 7, 2016
Dùng tỉ số kép để chứng minh thẳng hàng
Cho tam giác ABC và E, F thuộc AC, AB. M, N thuộc BE, CF. FM cắt EN tại T. BN cắt CM tại S. Chứng minh A, S, T thẳng hàng.
Dùng biến đổi góc để chứng minh thẳng hàng
Bài: Cho $\Delta ABC$ tù ở $A$ nội tiếp $(O),$ có đường cao $AH$ và đường trung bình ứng với $BC$ là $MN(M \in AB).$
$(HMN)$ cắt $(O)$ ở $P.PM,PN$ cắt $AC,AB$ ở $X,Y.$ Chứng minh $XY$ đi qua $H.$
Ta có $\angle HPM=\angle MNH=\angle MNA=\angle ACB\Longrightarrow P,X,H,C$ cùng thuộc một đường tròn
$\Longrightarrow \angle PHX=\angle PCA$. Tương tự thì $\angle PHY=\angle PBA\Longrightarrow \angle PHX=\angle PHY\Longrightarrow H,X,Y$ thẳng hàng hay $XY$ đi qua $H.\blacksquare$
$(HMN)$ cắt $(O)$ ở $P.PM,PN$ cắt $AC,AB$ ở $X,Y.$ Chứng minh $XY$ đi qua $H.$
Lời giải:
Ta có $\angle HPM=\angle MNH=\angle MNA=\angle ACB\Longrightarrow P,X,H,C$ cùng thuộc một đường tròn
$\Longrightarrow \angle PHX=\angle PCA$. Tương tự thì $\angle PHY=\angle PBA\Longrightarrow \angle PHX=\angle PHY\Longrightarrow H,X,Y$ thẳng hàng hay $XY$ đi qua $H.\blacksquare$
Ứng dụng đường tròn điểm trong giải toán
Đề:
Cho tam giác ABC và điểm P. AP, BP, CP cắt BC, CA, AB tại D, E, F. Gọi Q là điểm đẳng giác của P trong tam giác DEF. Đường thẳng qua P vuông góc với AP, BP, CP cắt BC, CA, AB tại X, Y, Z. Chứng minh X, Y, Z cùng nằm trên đường thẳng d và D vuông góc với PQ
Lời giải:
Dùng tỉ số kép với các đường lần lượt vuông góc như sau:
$P(AXYZ)=P(XABC)=A(XPBC)=A(PXCB)=A(PXYZ)$
Vậy X, Y, Z thẳng hàng.
Ý sau chứng minh khó theo Huỳnh Bách Khoa:
Ở lời giải trên ta đã dùng bổ đề sau:
Cho tam giác ABC và điểm P. AP, BP, CP cắt BC, CA, AB tại D, E, F. Gọi Q là điểm đẳng giác của P trong tam giác DEF. Đường thẳng qua P vuông góc với AP, BP, CP cắt BC, CA, AB tại X, Y, Z. Chứng minh X, Y, Z cùng nằm trên đường thẳng d và D vuông góc với PQ
Lời giải:
Dùng tỉ số kép với các đường lần lượt vuông góc như sau:
$P(AXYZ)=P(XABC)=A(XPBC)=A(PXCB)=A(PXYZ)$
Vậy X, Y, Z thẳng hàng.
Ý sau chứng minh khó theo Huỳnh Bách Khoa:
Thứ Hai, 11 tháng 7, 2016
Từ bổ đề đường tròn tiếp xúc đến bài toán mở rộng Serbia 2016
$\boxed{\text{Bài toán}}$(Serbia 2016) Cho tam giác $ABC$ có đường cao $AH$ và tâm nội tiếp $I$. $IB,IC$ cắt $AH$ tại $M,N$. $K,L$ là tâm ngoại tiếp tam giác $MAB,NAC$. $R,Q$ là trung điểm $IB,IC$. Lấy $P$ sao cho $PR\perp NL,PQ\perp MK$. Một tiếp tuyến thay đổi của đường tròn Euler của tam giác $IBC$ cắt $PQ,PR$ tại $S,T$. $X$ đối xứng $P$ qua $S,T$. Chứng minh rằng đường tròn $(XST)$ luôn tiếp xúc một đường tròn cố định khi tiếp tuyến thay đổi.
Lời giải:
$\boxed{\text{Bổ đề 1}}$ Cho tam giác $ABC$ có $O$ là tâm ngoại tiếp. Một đường tiếp tuyến với $\odot (BOC)$ cắt $AB$ ở $D$ và $CA$ ở $E$. Gọi $A'$ là điểm đối xứng của điểm $A$ qua $DE$. Khi đó $\odot (A'DE)$ tiếp xúc $\odot (ABC)$.
Chứng minh. Gọi $X$ là tiếp điểm $BX,CX$ cắt $\odot (O)$ tại $K,L.KE,LD$ cắt nhau tại $S$. Theo định lí $Pascal$ đảo thì $S$ thuộc $\odot (O)$
Biến đổi góc: $\angle DSE=\angle LCK=\angle BXC-\angle BKC=\angle BOC-\angle BAC=\angle BAC=\angle DA'E$
$\Longrightarrow D,S,A',E$ đồng viên.
Mặt khác: $\angle EXC=\angle KBC=\angle ESC$ nên $E,L,S,C$ đồng viên.
$\Longrightarrow E,X,S,C$ đồng viên.
$\Longrightarrow \angle DES=\angle XCS=\angle LKS$
$\Longrightarrow DE\parallel KL$
$\Longrightarrow \odot (DSE)$ tiếp xúc $\odot (DLK)$ hay $\odot (A'DE)$ tiếp xúc $\odot (ABC).\blacksquare$
$\boxed{\text{Bổ đề 2}}$ Cho tam giác $IBC.Q,R$ lần lượt là trung điểm $IC,IB$. Kí hiệu $\omega $ là đường tròn $Euler$ của $\triangle IBC.PR,PQ$ cắt $\omega $ tại $F,E$. Khi đó tâm $\odot (PEF)$ thuộc $\omega $.
Chứng minh. Gọi $I'$ là trung điểm $BC$.
Ta có $\angle RPE=180^\circ-\angle RIQ=180^\circ-\angle RI'Q=180^\circ-\angle REQ=\angle REP$
$\Longrightarrow \angle FRE=2\angle FPE\Longrightarrow $ tâm $\odot (PEF)$ thuộc $\omega .\blacksquare$
Quay lại bài toán.
Ta có $\angle IQP=90^\circ-\angle IMN+\angle AMK=90^\circ-\angle B/2+\angle C/2$. Tương tự $\angle IRP=90^\circ-\angle C/2+\angle B/2$.
Gọi $E,F$ lần lượt là giao của $PQ,PR$ với đường tròn $Euler$ của tam giác $IBC$. Theo bổ đề $2$ ta suy ra tâm $\odot (PEF)$ thuộc đường tròn $Euler$ của tam giác $IBC$. Theo bổ đề $1$ ta suy ra $\odot (PEF)$ tiếp xúc $\odot (XST).\blacksquare$
Kết hợp giữa phép nghịch đảo và định lý Miquel
Bài 1: Cho tam giác ABC, đường tròn (K) qua B, C cắt AC, AB tại E, F. BE cắt CF tại H. Gọi Q là tâm đường tròn (HEF), L là tâm đường tròn KBC. Chứng minh rằng Q, K, L thẳng hàng.
Lời giải
Gọi S là giao của (ABE) và (ACF) theo định lý Miquel ta có S thuộc (BFH) và (HEC). Suy ra:
∠BSC=360°-∠BSH-∠HSC=∠BFC+∠BEC=∠BKC Suy ra tứ giác BSKC nội tiếp.
Xét phép nghịch đảo tâm A đối với (K), biến:
F thành B
E thành C
Suy ra EB thành (ABE), CF thành (ACF)
Nên biến H là giao điểm EB và FC thành S là giao điểm (ABE), (ACF)
Nên phép vị tự này biến (HEF) thành (BSC) nên tồn tại phép vị tự tâm A tỉ số k biến Q thành L.
Vậy A, Q, L thẳng hàng
Bài 2: (tiếng anh) Let $\triangle ABC$. A circle passes through $B,C$ intersects $AC,AB$ at $E,F$. The lines passes through $E,F$ and perpendicular to $AC,AB$ intersect together at $O. M,N$ lies on $EO,FO$,respectively. Draw $CQ\perp AM,BP\perp AN. BP$ and $AM$ intersects together at $I.MB$ intersect $NC$ at $K$.
Prove that $O,I,K$ are collinear.
Solution
Lemma: Let $ABCD$ is a quadrilateral. $AB$ cuts $CD$ at $G$; $AD$ cuts $BC$ at $K$. Then three circles with diameter $AC,BD,GK$ have the same radical axis.
Proof
Let $H,H'$ are the orthocenters of $\triangle GAD$ and $\triangle KCD$ respectively. Suppose that $GH,AH,DH$ cut $AD,GD,GA$ at $P,Q,R$ respectively. We have $HG.HP=HA.HQ=HD.HR$ so $H$ is on the radical axis of three circles with diameter $AC,BD,GK$. Similarly, we have $H'$ is on the radical axis of three circles with diameter $AC,BD,GK$, too. Hence, $HH'$ is the radical axis of three circles with diameter $AC,BD,GK$ so three cicles with diameter $AC,BD,GK$ have the same radical axis.
Back to this problem Let $X,Y,Z$ are the midpoints of $AO,AI,AK$ respectively. $BC$ cuts $MN$ at $G$. $H$ lies on $AK$ such that $GH \perp AK$. We have $AF.AB=AE.AC$ so $A$ lies on the radical axis of the circles with diameter $BN,CM$. According this lemma, we have $AH.AK=AF.AB=AE.AC$. The inversion center $A$, radius $AF.AB$ : $(X;XA) \mapsto BC; (Y;YA) \mapsto MN; (Z,ZA) \mapsto$ the line passing $H$ and perpendicular $AK$. But $BC, MN$ and the line passing $H$ and perpendicular $AK$ are concurrent at $G$. So $(X;XA), (Y;YA), (Z;ZA)$ have the same radical axis $\Longrightarrow$ $X,Y,Z$ are collinear. Hence $O,I,K$ are collinear.
Bài 3: Cho tam giác ABC. Đường tròn (O) đi qua B, C cắt AC, AB tại E, F. BE cắt CF tại D. H là hình chiếu của O trên AD. K, L là tâm (AFC) và (AEB). I là giao điểm khác H của (KHF) và (LHE). CMR: AI đi qua trung điểm BC.
Lời giải (Huỳnh Bách Khoa)
Lưu ý thêm:
Ta cần bổ đề sau: Gọi H là giao của (BDF) và (DEC). Thì OH vuông DH. Ta có thể gọi M, N là trung điểm FC, EB. Sau đó dùng vị tự quay để suy ra OHMND đồng viên đường tròn đường kính OD. (China 1992)
Và một số sai sót: M thuộc (LEH) do ∠MLH=∠HEC
Phép gọi T là đối xứng D qua trung điểm BC thường hay gặp để tạo ra đường đẳng giác góc A với AH
Và còn ∠DHN=∠NAH=∠DXN
Về phần phép nghịch đảo là phương tích AE.AC, biến E thành C, F thành B. Thay điểm G thành D. Do I là giao của (MEH) và (NHF) nên biến thành Z là giao của (BDX) và (CDY)
Lời giải
∠BSC=360°-∠BSH-∠HSC=∠BFC+∠BEC=∠BKC Suy ra tứ giác BSKC nội tiếp.
Xét phép nghịch đảo tâm A đối với (K), biến:
F thành B
E thành C
Suy ra EB thành (ABE), CF thành (ACF)
Nên biến H là giao điểm EB và FC thành S là giao điểm (ABE), (ACF)
Nên phép vị tự này biến (HEF) thành (BSC) nên tồn tại phép vị tự tâm A tỉ số k biến Q thành L.
Vậy A, Q, L thẳng hàng
Bài 2: (tiếng anh) Let $\triangle ABC$. A circle passes through $B,C$ intersects $AC,AB$ at $E,F$. The lines passes through $E,F$ and perpendicular to $AC,AB$ intersect together at $O. M,N$ lies on $EO,FO$,respectively. Draw $CQ\perp AM,BP\perp AN. BP$ and $AM$ intersects together at $I.MB$ intersect $NC$ at $K$.
Prove that $O,I,K$ are collinear.
Solution
Lemma: Let $ABCD$ is a quadrilateral. $AB$ cuts $CD$ at $G$; $AD$ cuts $BC$ at $K$. Then three circles with diameter $AC,BD,GK$ have the same radical axis.
Proof
Let $H,H'$ are the orthocenters of $\triangle GAD$ and $\triangle KCD$ respectively. Suppose that $GH,AH,DH$ cut $AD,GD,GA$ at $P,Q,R$ respectively. We have $HG.HP=HA.HQ=HD.HR$ so $H$ is on the radical axis of three circles with diameter $AC,BD,GK$. Similarly, we have $H'$ is on the radical axis of three circles with diameter $AC,BD,GK$, too. Hence, $HH'$ is the radical axis of three circles with diameter $AC,BD,GK$ so three cicles with diameter $AC,BD,GK$ have the same radical axis.
Back to this problem Let $X,Y,Z$ are the midpoints of $AO,AI,AK$ respectively. $BC$ cuts $MN$ at $G$. $H$ lies on $AK$ such that $GH \perp AK$. We have $AF.AB=AE.AC$ so $A$ lies on the radical axis of the circles with diameter $BN,CM$. According this lemma, we have $AH.AK=AF.AB=AE.AC$. The inversion center $A$, radius $AF.AB$ : $(X;XA) \mapsto BC; (Y;YA) \mapsto MN; (Z,ZA) \mapsto$ the line passing $H$ and perpendicular $AK$. But $BC, MN$ and the line passing $H$ and perpendicular $AK$ are concurrent at $G$. So $(X;XA), (Y;YA), (Z;ZA)$ have the same radical axis $\Longrightarrow$ $X,Y,Z$ are collinear. Hence $O,I,K$ are collinear.
Bài 3: Cho tam giác ABC. Đường tròn (O) đi qua B, C cắt AC, AB tại E, F. BE cắt CF tại D. H là hình chiếu của O trên AD. K, L là tâm (AFC) và (AEB). I là giao điểm khác H của (KHF) và (LHE). CMR: AI đi qua trung điểm BC.
Lời giải (Huỳnh Bách Khoa)
Lưu ý thêm:
Ta cần bổ đề sau: Gọi H là giao của (BDF) và (DEC). Thì OH vuông DH. Ta có thể gọi M, N là trung điểm FC, EB. Sau đó dùng vị tự quay để suy ra OHMND đồng viên đường tròn đường kính OD. (China 1992)
Và một số sai sót: M thuộc (LEH) do ∠MLH=∠HEC
Phép gọi T là đối xứng D qua trung điểm BC thường hay gặp để tạo ra đường đẳng giác góc A với AH
Và còn ∠DHN=∠NAH=∠DXN
Về phần phép nghịch đảo là phương tích AE.AC, biến E thành C, F thành B. Thay điểm G thành D. Do I là giao của (MEH) và (NHF) nên biến thành Z là giao của (BDX) và (CDY)
Dùng đạo hàm để giải bất đẳng thức
Đề:
Cho a, b, c là các số thực không âm đôi một khác nhau
Tìm giá trị nhỏ nhất của biểu thức $P=\left [ (a+b)^{2}+(b+c)^{2}+(c+a)^{2} \right ]\left [ \frac{1}{(a-b)^{2}}+\frac{1}{(b-c)^{2}}+\frac{1}{(c-a)^{2}} \right ]$
Lời giải:
Giả sử $a > b > c \ge 0$
Khi đó $P= \Big[(a+b)^2+(b+c)^2+(c+a)^2 \Big].\left[ \frac{1}{(a-b)^2}+\frac{1}{(b-c)^2}+\frac{1}{(a-c)^2}\right]$
$\ge \left[ (a+b)^2 +a^2+b^2\right] .\left[ \frac{1}{(a-b)^2}+\frac{1}{a^2}+\frac{1}{b^2}\right]$
$=2\left[(a+b)^2-ab \right].\left[ \frac{1}{(a+b)^2-4ab}+\frac{(a+b)^2}{a^2b^2}-\frac{2}{ab}\right]$
BDT đã cho có dạng thuần nhất nên chuẩn hóa $a+b=1,$ đặt $ab=x,0< x \le \frac 14$
Khi đó $\frac F2 \ge(1-x)\left( \frac{1}{1-4x}+\frac{1}{x^2}-\frac 2x\right)$
$=3+\frac{1}{x^2}-\frac{3}{x}+\frac{3x}{1-4x}$
Khảo sát hàm số $f(x)=\frac{1}{x^2}-\frac{3}{x}+\frac{3x}{1-4x}$ trên $\left[0;\frac 14 \right)$
Ta thu được $\min f(x)=\frac{35+11\sqrt{33}}{8}$
Từ đó suy ra $F \ge \frac{59+11\sqrt{33}}{4}$
$\min F=\frac{59+11\sqrt{33}}{4}$ đạt được chẳng hạn khi $a,b$ là 2 nghiệm của pt $x^2-x+\frac{13-\sqrt{33}}{34},c=0$
Cho a, b, c là các số thực không âm đôi một khác nhau
Tìm giá trị nhỏ nhất của biểu thức $P=\left [ (a+b)^{2}+(b+c)^{2}+(c+a)^{2} \right ]\left [ \frac{1}{(a-b)^{2}}+\frac{1}{(b-c)^{2}}+\frac{1}{(c-a)^{2}} \right ]$
Lời giải:
Giả sử $a > b > c \ge 0$
Khi đó $P= \Big[(a+b)^2+(b+c)^2+(c+a)^2 \Big].\left[ \frac{1}{(a-b)^2}+\frac{1}{(b-c)^2}+\frac{1}{(a-c)^2}\right]$
$\ge \left[ (a+b)^2 +a^2+b^2\right] .\left[ \frac{1}{(a-b)^2}+\frac{1}{a^2}+\frac{1}{b^2}\right]$
$=2\left[(a+b)^2-ab \right].\left[ \frac{1}{(a+b)^2-4ab}+\frac{(a+b)^2}{a^2b^2}-\frac{2}{ab}\right]$
BDT đã cho có dạng thuần nhất nên chuẩn hóa $a+b=1,$ đặt $ab=x,0< x \le \frac 14$
Khi đó $\frac F2 \ge(1-x)\left( \frac{1}{1-4x}+\frac{1}{x^2}-\frac 2x\right)$
$=3+\frac{1}{x^2}-\frac{3}{x}+\frac{3x}{1-4x}$
Khảo sát hàm số $f(x)=\frac{1}{x^2}-\frac{3}{x}+\frac{3x}{1-4x}$ trên $\left[0;\frac 14 \right)$
Ta thu được $\min f(x)=\frac{35+11\sqrt{33}}{8}$
Từ đó suy ra $F \ge \frac{59+11\sqrt{33}}{4}$
$\min F=\frac{59+11\sqrt{33}}{4}$ đạt được chẳng hạn khi $a,b$ là 2 nghiệm của pt $x^2-x+\frac{13-\sqrt{33}}{34},c=0$
Bài toán mở rộng về đường tròn chín điểm ( Tiếng Anh)
Let $ABC$ be a triangle with circumcircle $(O)$. $(K)$ is a circle passing through $B,C$. $(K)$ cuts $CA,AB$ again at $E,F$. $BE$ cuts $CF$ at $H_K$.
a) Prove that $H_KK$ and $AO$ intersect on $(O)$.
b) $O_K$ is isogonal conjugate of $H_K$ with respect to triangle $ABC$. Prove that $O_K$ lies on $OK$.
c) Let $L,N$ be the points on $CA,AB$, resp such that $O_KL\parallel BE, O_KN\parallel CF$. Prove that $LN\parallel BC$.
d) The line passing through $N$ parallel to $BE$ cuts the line passing through $L$ parallel to $CF$ at $P$. Prove that $P$ lies on $AH_K$.
e) $Q,R$ lie on $BE,CF$, resp such that $PQ\parallel AB,PR\parallel AC$. Prove that $QR\parallel BC$.
f) Prove that $NQ,LR$ and $AH_K$ are concurrent.
g) $D$ is projection of $K$ on $AH_K$. Prove that $DK,EF,BC$ are concurrent.
h) Prove that $KN\perp BE, KL\perp CF$.
i) Prove that nine points $D,E,F;P,Q,R;K,L,N$ lie on a circle $(N_K)$.
j) Prove that $N_K$ is midpoint of $PK$ and $KN_K$ is parallel to $AO$.
k) Prove that $H_K,N_K,O$ are collinear.
When $K \equiv M$ midpoint of $BC$, we get all properities of Nine-point circle.
Solution:
a, g) Let $S \equiv EF \cap BC.$ Then $AS$ is the polar of $H_K$ WRT $(K)$ and $AH_K$ is the polar of $S$ WRT $(K)$ $\Longrightarrow$ $KH_K$ is perpendicular to $AS$ through $H$ and $AH_K$ is perpendicular to $KS$ through $D.$ Hence $SH \cdot SA=SD \cdot SK=SB \cdot SC$ $\Longrightarrow$ $H \in (O).$ Since $\angle AHH_K=90^{\circ},$ then $KH_K$ and $AO$ meet on $(O).$
b, c) $\angle O_KBC=\angle H_KBF=\angle H_KCE=\angle O_KCB$ $\Longrightarrow$ $O_K$ is on perpendicular bisector $OK$ of $\overline{BC}.$ $\angle BFC=\angle BNO_K=\angle BKO_K$ (mod 180) $\Longrightarrow$ $N,B,K,O_K$ are concyclic $\Longrightarrow$ $\angle BNK=\angle BO_KK.$ But $\angle BO_KK=90^{\circ}-\angle FBE=\angle FEK$ $\Longrightarrow$ $\angle BNK=\angle FEK,$ i.e. $N$ lies on circumcircle $(N_K)$ of $DKEF.$ Similarly, $L \in (N_K).$ Thus, $LN$ is antiparallel to $EF$ WRT $AE,AF$ $\Longrightarrow$ $LN \parallel BC.$
d, f)$\triangle PLN$ and $\triangle H_KCB,$ with parallel sides, are homothetic with center $A$ $\Longrightarrow$ $A,P,H_K$ are collinear. Likewise, $\triangle ANL$ and $\triangle PQR,$ with parallel sides, are homothetic with center $AP \cap NQ \cap LR,$ i.e. $AH_K,NQ,LR$ concur.
i) $\angle NKL=\angle NBO_K+\angle LCO_K=CBH_K+\angle BCH_K=\angle NPL$ (mod 180) $\Longrightarrow$ $P \in (N_K).$ Further, $P$ is the midpoint of the arc $EF$ of $(N_K),$ because $\angle PFK=\angle PDK=90^{\circ},$ i.e. $KP$ is perpendicular bisector of $\overline{EF}.$ Now, since $\angle PQE=\angle FBE=\angle PKE,$ it follows that $Q \in (N_K).$ Similarly, $R \in (N_K).$
h, j) $D,E,F,P,Q,R,K,L,N$ lie then on a circle $(N_K)$ with diameter $KP$ perpendicular to $EF,$ i.e. $KP \parallel AO.$ Thus, $KN$ is perpendicular to $PN \parallel BE$ and $KL$ is perpendicular to $PL \parallel CF.$
e, k) $\triangle PQR \cup (N_K)$ and $\triangle ABC \cup (O)$ are homothetic with center $H_K \equiv AP \cap BQ \cap CR,$ thus $QR \parallel BC$ and $H_K,O,N_K$ are collinear.
a) Prove that $H_KK$ and $AO$ intersect on $(O)$.
b) $O_K$ is isogonal conjugate of $H_K$ with respect to triangle $ABC$. Prove that $O_K$ lies on $OK$.
c) Let $L,N$ be the points on $CA,AB$, resp such that $O_KL\parallel BE, O_KN\parallel CF$. Prove that $LN\parallel BC$.
d) The line passing through $N$ parallel to $BE$ cuts the line passing through $L$ parallel to $CF$ at $P$. Prove that $P$ lies on $AH_K$.
e) $Q,R$ lie on $BE,CF$, resp such that $PQ\parallel AB,PR\parallel AC$. Prove that $QR\parallel BC$.
f) Prove that $NQ,LR$ and $AH_K$ are concurrent.
g) $D$ is projection of $K$ on $AH_K$. Prove that $DK,EF,BC$ are concurrent.
h) Prove that $KN\perp BE, KL\perp CF$.
i) Prove that nine points $D,E,F;P,Q,R;K,L,N$ lie on a circle $(N_K)$.
j) Prove that $N_K$ is midpoint of $PK$ and $KN_K$ is parallel to $AO$.
k) Prove that $H_K,N_K,O$ are collinear.
When $K \equiv M$ midpoint of $BC$, we get all properities of Nine-point circle.
Solution:
a, g) Let $S \equiv EF \cap BC.$ Then $AS$ is the polar of $H_K$ WRT $(K)$ and $AH_K$ is the polar of $S$ WRT $(K)$ $\Longrightarrow$ $KH_K$ is perpendicular to $AS$ through $H$ and $AH_K$ is perpendicular to $KS$ through $D.$ Hence $SH \cdot SA=SD \cdot SK=SB \cdot SC$ $\Longrightarrow$ $H \in (O).$ Since $\angle AHH_K=90^{\circ},$ then $KH_K$ and $AO$ meet on $(O).$
b, c) $\angle O_KBC=\angle H_KBF=\angle H_KCE=\angle O_KCB$ $\Longrightarrow$ $O_K$ is on perpendicular bisector $OK$ of $\overline{BC}.$ $\angle BFC=\angle BNO_K=\angle BKO_K$ (mod 180) $\Longrightarrow$ $N,B,K,O_K$ are concyclic $\Longrightarrow$ $\angle BNK=\angle BO_KK.$ But $\angle BO_KK=90^{\circ}-\angle FBE=\angle FEK$ $\Longrightarrow$ $\angle BNK=\angle FEK,$ i.e. $N$ lies on circumcircle $(N_K)$ of $DKEF.$ Similarly, $L \in (N_K).$ Thus, $LN$ is antiparallel to $EF$ WRT $AE,AF$ $\Longrightarrow$ $LN \parallel BC.$
d, f)$\triangle PLN$ and $\triangle H_KCB,$ with parallel sides, are homothetic with center $A$ $\Longrightarrow$ $A,P,H_K$ are collinear. Likewise, $\triangle ANL$ and $\triangle PQR,$ with parallel sides, are homothetic with center $AP \cap NQ \cap LR,$ i.e. $AH_K,NQ,LR$ concur.
i) $\angle NKL=\angle NBO_K+\angle LCO_K=CBH_K+\angle BCH_K=\angle NPL$ (mod 180) $\Longrightarrow$ $P \in (N_K).$ Further, $P$ is the midpoint of the arc $EF$ of $(N_K),$ because $\angle PFK=\angle PDK=90^{\circ},$ i.e. $KP$ is perpendicular bisector of $\overline{EF}.$ Now, since $\angle PQE=\angle FBE=\angle PKE,$ it follows that $Q \in (N_K).$ Similarly, $R \in (N_K).$
h, j) $D,E,F,P,Q,R,K,L,N$ lie then on a circle $(N_K)$ with diameter $KP$ perpendicular to $EF,$ i.e. $KP \parallel AO.$ Thus, $KN$ is perpendicular to $PN \parallel BE$ and $KL$ is perpendicular to $PL \parallel CF.$
e, k) $\triangle PQR \cup (N_K)$ and $\triangle ABC \cup (O)$ are homothetic with center $H_K \equiv AP \cap BQ \cap CR,$ thus $QR \parallel BC$ and $H_K,O,N_K$ are collinear.
Thứ Bảy, 9 tháng 7, 2016
Dùng pqr để chứng minh bất đẳng thức
Bài 1:
Cho ba số thực
a⩾b⩾1⩾c⩾0a⩾b⩾1⩾c⩾0 thỏa mãn điều kiện a+b+c=3.
Chứng minh rằng
$\frac{24}{a^3+b^3+c^3} + \frac{25}{ab+bc+ca} \geqslant 14.$ Đẳng thức xảy ra khi nào ?
Lời giải:
Đặt $q=ab+bc+ca, r=abc.$
$1.$ Vì $(1-a)(1-b)(1-c)=q-r-2\geq 0$ nên $q\geq r+2\geq 2.$
Lại có $3(ab+bc+ca)\leq (a+b+c)^2$ nên $ab+bc+ca\leq 3.$
$2.$ Bất đẳng thức tương đương
$$\frac{8}{9-3q+r}+\frac{25}{q}\geq 14$$
mà ta lại có $r\leq q-2$ nên ta chỉ cần chứng minh
$$\frac{8}{7-2q}+\frac{25}{q}\geq 14$$
$$\Leftrightarrow 7(2q-5)^2\geq 0.$$
Đẳng thức xảy ra khi và chỉ khi $a=\frac{2+\sqrt{2}}{2}, b=1, c=\frac{2-\sqrt{2}}{2}.$
Bài 2:
$x,y,z >0$ và $x^2+y^2+z^2=3$.CMR: $\sum \frac{1}{7-xy} \leq \frac{1}{2}$
Lời giải:
Bất đẳng thức tương đương
\[294-28(xy+yz+xz)+2xyz(x+y+z)\leq 343-49(xy+yz+xz)+7xyz(x+y+z)-(xyz)^2\]
\[\Leftrightarrow 21(xy+yz+xz)+(xyz)^2\leq 49+5xyz(x+y+z)\]
Đặt $x+y+z=p,xy+yz+xz=q,xyz=r$ và $p^2-2q=3$, Ta cần chứng minh
\[21q+r^2\leq 49+ 5pr\]
Vì $r^2\leq 1$ Ta sẽ chứng minh bất đẳng thức mạnh hơn
\[21q\leq 48+5pr\]
Theo bất đẳng thức Schur bậc 3, $r\geq \frac{p(4q-p^2)}{9}=\frac{p(2q-3)}{9}$, Ta cần chứng minh rằng
\[21q\leq 48+\frac{5p^2(2q-3)}{9}=\frac{20q^2-45}{9}\]
\[\Leftrightarrow (q-3)(q-\frac{129}{20})\geq 0 \ \text{(Luôn đúng)} \ q\leq 3\].
Dấu bằng xảy ra khi và chỉ khi $a=b=c=1$.
Cho ba số thực
a⩾b⩾1⩾c⩾0a⩾b⩾1⩾c⩾0 thỏa mãn điều kiện a+b+c=3.
Chứng minh rằng
$\frac{24}{a^3+b^3+c^3} + \frac{25}{ab+bc+ca} \geqslant 14.$ Đẳng thức xảy ra khi nào ?
Lời giải:
Đặt $q=ab+bc+ca, r=abc.$
$1.$ Vì $(1-a)(1-b)(1-c)=q-r-2\geq 0$ nên $q\geq r+2\geq 2.$
Lại có $3(ab+bc+ca)\leq (a+b+c)^2$ nên $ab+bc+ca\leq 3.$
$2.$ Bất đẳng thức tương đương
$$\frac{8}{9-3q+r}+\frac{25}{q}\geq 14$$
mà ta lại có $r\leq q-2$ nên ta chỉ cần chứng minh
$$\frac{8}{7-2q}+\frac{25}{q}\geq 14$$
$$\Leftrightarrow 7(2q-5)^2\geq 0.$$
Đẳng thức xảy ra khi và chỉ khi $a=\frac{2+\sqrt{2}}{2}, b=1, c=\frac{2-\sqrt{2}}{2}.$
Bài 2:
$x,y,z >0$ và $x^2+y^2+z^2=3$.CMR: $\sum \frac{1}{7-xy} \leq \frac{1}{2}$
Lời giải:
Bất đẳng thức tương đương
\[294-28(xy+yz+xz)+2xyz(x+y+z)\leq 343-49(xy+yz+xz)+7xyz(x+y+z)-(xyz)^2\]
\[\Leftrightarrow 21(xy+yz+xz)+(xyz)^2\leq 49+5xyz(x+y+z)\]
Đặt $x+y+z=p,xy+yz+xz=q,xyz=r$ và $p^2-2q=3$, Ta cần chứng minh
\[21q+r^2\leq 49+ 5pr\]
Vì $r^2\leq 1$ Ta sẽ chứng minh bất đẳng thức mạnh hơn
\[21q\leq 48+5pr\]
Theo bất đẳng thức Schur bậc 3, $r\geq \frac{p(4q-p^2)}{9}=\frac{p(2q-3)}{9}$, Ta cần chứng minh rằng
\[21q\leq 48+\frac{5p^2(2q-3)}{9}=\frac{20q^2-45}{9}\]
\[\Leftrightarrow (q-3)(q-\frac{129}{20})\geq 0 \ \text{(Luôn đúng)} \ q\leq 3\].
Dấu bằng xảy ra khi và chỉ khi $a=b=c=1$.
Từ đẳng thức cho đến bất đẳng thức
Ta có đẳng thức sau:
Cho $xyz=1$ thì khi đó:
$\sum \frac{1}{xy+x+1}=1$
Ứng dụng:
Bài 1; (APMO 2016)
Cho ba số dương x,y,zx,y,z thỏa mãn điều kiện xyz=1. Chứng minh rằng
$\frac{1}{(x+1)^2+y^2+1} + \frac{1}{(y+1)^2+z^2+1} + \frac{1}{(z+1)^2+x^2+1} \leqslant {\frac{1}{2}}.$
Áp dụng $AM-GM:$
$$\sum \frac{1}{(x+1)^2+y^2+1}=\sum \frac{1}{x^2+2x+1+y^2+1}\leq \frac{1}{2}\sum \frac{1}{xy+x+1}=\frac{1}{2}(\text{do }xyz=1)$$
Đẳng thức xảy ra khi $x=y=z=1.$
Bài 2 (Romanian JBMO TST 2016). Với a,b,ca,b,c là ba số thực dương thỏa mãn điều kiện abc⩾1. Chứng minh rằng:
Vì $abc \geq 1$ nên tồn tại $k \geq 1$ và $a',b',c'>0$ sao cho $a=ka', b=kb', c=kc'$ và $a'b'c'=1$.
Suy ra $a\geq a', b\geq b', c\geq c'$.
Do đó
\[\sum_{a,b,c}\frac{1}{a^3+2b^3+6}\leq \sum_{a',b',c'}\frac{1}{a'^3+2b'^3+6}=\sum_{a',b',c'}\frac{1}{(a'^3+b'^3+1)+(b'^3+1+1)+3}\leq \frac{1}{3}\sum_{a',b',c'}\frac{1}{a'b'+b'+1}=\frac{1}{3}.\]
Ta có đpcm.
Cho $xyz=1$ thì khi đó:
$\sum \frac{1}{xy+x+1}=1$
Ứng dụng:
Bài 1; (APMO 2016)
Cho ba số dương x,y,zx,y,z thỏa mãn điều kiện xyz=1. Chứng minh rằng
Lời giải:
Áp dụng $AM-GM:$
$$\sum \frac{1}{(x+1)^2+y^2+1}=\sum \frac{1}{x^2+2x+1+y^2+1}\leq \frac{1}{2}\sum \frac{1}{xy+x+1}=\frac{1}{2}(\text{do }xyz=1)$$
Đẳng thức xảy ra khi $x=y=z=1.$
$\dfrac{1}{a^3+2b^3+6}+\dfrac{1}{b^3+2c^3+6}+\dfrac{1}{c^3+2a^3+6} \le \dfrac{1}{3}.$
Lời giải:
Vì $abc \geq 1$ nên tồn tại $k \geq 1$ và $a',b',c'>0$ sao cho $a=ka', b=kb', c=kc'$ và $a'b'c'=1$.
Suy ra $a\geq a', b\geq b', c\geq c'$.
Do đó
\[\sum_{a,b,c}\frac{1}{a^3+2b^3+6}\leq \sum_{a',b',c'}\frac{1}{a'^3+2b'^3+6}=\sum_{a',b',c'}\frac{1}{(a'^3+b'^3+1)+(b'^3+1+1)+3}\leq \frac{1}{3}\sum_{a',b',c'}\frac{1}{a'b'+b'+1}=\frac{1}{3}.\]
Ta có đpcm.
Ứng dụng bất đẳng thức Bernoulli
Cho n số thực dương a1,a2,…,an. Chứng minh rằng;
$\prod_{i=1}^{n} \left(1+\frac{1}{a_i}\right)^{a_{i+1}-a_i} \geqslant 1,$
Lời giải:
Áp dụng BĐT $Bernoulli,$ ta có:
$$\left(1+\frac{1}{a_i}\right)^{a_{i+1}-a_i}\geq 1+\frac{a_{i+1}-a_i}{a_i}=\frac{a_{i+1}}{a_i}$$
Do đó:
$$\prod_{i=1}^{n} \left(1+\frac{1}{a_i}\right)^{a_{i+1}-a_i}\geq \frac{a_2}{a_1}.\frac{a_3}{a_2}\cdots \frac{a_1}{a_n}=1$$
$\prod_{i=1}^{n} \left(1+\frac{1}{a_i}\right)^{a_{i+1}-a_i} \geqslant 1,$
Lời giải:
Áp dụng BĐT $Bernoulli,$ ta có:
$$\left(1+\frac{1}{a_i}\right)^{a_{i+1}-a_i}\geq 1+\frac{a_{i+1}-a_i}{a_i}=\frac{a_{i+1}}{a_i}$$
Do đó:
$$\prod_{i=1}^{n} \left(1+\frac{1}{a_i}\right)^{a_{i+1}-a_i}\geq \frac{a_2}{a_1}.\frac{a_3}{a_2}\cdots \frac{a_1}{a_n}=1$$
Đánh giá từng số hạng trong bất đẳng thức
Đề: Cho ba số thực dương a,b,ca,b,c thỏa mãn điều kiện a+b+c=3. Chứng minh rằng
$\frac{a^3+b^3}{a^2+ab+b^2}+\frac{b^3+c^3}{b^2+bc+c^2}+\frac{c^3+a^3}{c^2+ca+a^2} \geqslant 2.$
Lời giải:
Ta có:
$$\frac{a^2-ab+b^2}{a^2+ab+b^2}\geq \frac{1}{3}\Leftrightarrow \frac{2(a-b)^2}{3(a^2+ab+b^2)}\geq 0,\text{đúng}$$
Do đó:
$$\sum \frac{a^3+b^3}{a^2+ab+b^2}=\sum (a+b).\frac{a^2-ab+b^2}{a^2+ab+b^2}\geq \sum \frac{1}{3}(a+b)=2$$
Đẳng thức xảy ra khi $a=b=c=1$
Đặt ẩn phụ trong chứng minh bất đẳng thức
Đề bài: Cho ba số thực dương
a,b,c thỏa mãn a+b+c=3. Chứng minh rằng
$\sqrt{\frac{b}{a^2+3}}+\sqrt{\frac{c}{b^2+3}}+\sqrt{\frac{a}{c^2+3}} \leqslant \frac32\sqrt[4]{\frac{1}{abc}}.$
Lời giải
Ta sẽ cố gắng đưa về bất đẳng thức đồng bậc
Vế phải của bất đẳng thức thực ra là $\dfrac{1}{2}.\dfrac{3}{\sqrt[4]{abc}}=\dfrac{a+b+c}{2\sqrt[4]{abc}}$
Áp dụng bất đẳng thức AM-GM thì $a^2+3\geq 4\sqrt{a}$ nên $VT\leq \sum \dfrac{\sqrt{b}}{2\sqrt[4]{a}}$
Đặt $\sqrt[4]{a}=x,\cdots $ thì ta chỉ cần chứng minh $\sum \dfrac{y^2}{x}\leq \dfrac{x^4+y^4+z^4}{xyz}\Leftrightarrow x^3y+y^3z+z^3x\leq x^4+y^4+z^4$
Bất đẳng thức trên chứng minh khá dễ bằng AM-GM
a,b,c thỏa mãn a+b+c=3. Chứng minh rằng
$\sqrt{\frac{b}{a^2+3}}+\sqrt{\frac{c}{b^2+3}}+\sqrt{\frac{a}{c^2+3}} \leqslant \frac32\sqrt[4]{\frac{1}{abc}}.$
Lời giải
Ta sẽ cố gắng đưa về bất đẳng thức đồng bậc
Vế phải của bất đẳng thức thực ra là $\dfrac{1}{2}.\dfrac{3}{\sqrt[4]{abc}}=\dfrac{a+b+c}{2\sqrt[4]{abc}}$
Áp dụng bất đẳng thức AM-GM thì $a^2+3\geq 4\sqrt{a}$ nên $VT\leq \sum \dfrac{\sqrt{b}}{2\sqrt[4]{a}}$
Đặt $\sqrt[4]{a}=x,\cdots $ thì ta chỉ cần chứng minh $\sum \dfrac{y^2}{x}\leq \dfrac{x^4+y^4+z^4}{xyz}\Leftrightarrow x^3y+y^3z+z^3x\leq x^4+y^4+z^4$
Bất đẳng thức trên chứng minh khá dễ bằng AM-GM
Điểm rơi trong bất đẳng thức Cauchy
(Saudi Arabia TST 2016). Cho ba số thực dương a,b,c thỏa mãn điều kiện a+b+c=3. Chứng minh rằng:
$\frac{1}{a^2+b^2+c^2}+\frac{1}{\sqrt{abc}} \geqslant \frac{4}{3}.$
Lời giải:
Đặt $t=ab+bc+ca$, với $t \in (0;3]$.
Sử dụng liên tiếp AM-GM ta có $3^6 = [a^2+b^2+c^2+2(ab+bc+ca)]^3 \geq 27(a^2+b^2+c^2)(ab+bc+ca)^2$ $\Leftrightarrow$ $\frac{1}{a^2+b^2+c^2} \geq \frac{t^2}{27}$. Mặt khác $3 \geq t \geq 3 \sqrt[3]{a^2b^2c^2}$. Do đó $\frac{1}{\sqrt{abc}} \geq \sqrt{\dfrac{27}{t^3}}$. Từ đó ta có $\frac{1}{a^2+b^2+c^2} + \frac{1}{\sqrt{abc}} \geq \frac{t^2}{27} + \sqrt{\frac{27}{t^3}} = \frac{t^2}{81}.3 + \frac{1}{9}\sqrt{\frac{27}{t^3}}.4 + \dfrac{5}{9}\sqrt{\frac{27}{t^3}} \geq 7\sqrt[7]{(\frac{t^2}{81})^3.(\dfrac{1}{9}\sqrt{\frac{27}{t^3}})^4} + \frac{5}{9} = \frac{4}{3}$. Dấu bằng xảy ra khi $a=b=c=1$.
$\frac{1}{a^2+b^2+c^2}+\frac{1}{\sqrt{abc}} \geqslant \frac{4}{3}.$
Lời giải:
Đặt $t=ab+bc+ca$, với $t \in (0;3]$.
Sử dụng liên tiếp AM-GM ta có $3^6 = [a^2+b^2+c^2+2(ab+bc+ca)]^3 \geq 27(a^2+b^2+c^2)(ab+bc+ca)^2$ $\Leftrightarrow$ $\frac{1}{a^2+b^2+c^2} \geq \frac{t^2}{27}$. Mặt khác $3 \geq t \geq 3 \sqrt[3]{a^2b^2c^2}$. Do đó $\frac{1}{\sqrt{abc}} \geq \sqrt{\dfrac{27}{t^3}}$. Từ đó ta có $\frac{1}{a^2+b^2+c^2} + \frac{1}{\sqrt{abc}} \geq \frac{t^2}{27} + \sqrt{\frac{27}{t^3}} = \frac{t^2}{81}.3 + \frac{1}{9}\sqrt{\frac{27}{t^3}}.4 + \dfrac{5}{9}\sqrt{\frac{27}{t^3}} \geq 7\sqrt[7]{(\frac{t^2}{81})^3.(\dfrac{1}{9}\sqrt{\frac{27}{t^3}})^4} + \frac{5}{9} = \frac{4}{3}$. Dấu bằng xảy ra khi $a=b=c=1$.
Phương pháp sắp thứ tự các biến trong bất đẳng thức
Bài (Russia 2016): Cho bốn số thực dương a,b,c,d thỏa mãn điều kiện a+b+c+d=3. Chứng minh rằng:
a) $\frac{1}{a^2}+\frac{1}{b^2}+\frac{1}{c^2}+\frac{1}{d^2}\le\frac{1}{a^2b^2c^2d^2},$
b) $\frac{1}{a^3}+\frac{1}{b^3}+\frac{1}{c^3}+\frac{1}{d^3}\le\frac{1}{a^3b^3c^3d^3}.$
Lời giải
$(a)$ Giả sử $a \geqslant b \geqslant c \geqslant d$, BĐT cần chứng minh tương đương với
$a^{2}b^{2}c^{2}+a^{2}b^{2}d^{2}+a^{2}c^{2}d^{2}+b^{2}c^{2}d^{2} \leqslant 1$
Ta đưa BĐT về dạng đồng bậc
$a^{2}b^{2}c^{2}+a^{2}b^{2}d^{2}+a^{2}c^{2}d^{2}+b^{2}c^{2}d^{2} \leqslant \frac{1}{3^6}(a+b+c)^6$
Do $a \geqslant b \geqslant c \geqslant d$ nên ta có
\[\begin{aligned} a^{2}b^{2}c^{2}+a^{2}b^{2}d^{2}+a^{2}c^{2}d^{2}+b^{2}c^{2}d^{2} &\leqslant a^{2}b^{2}c^{2}+a^{2}b^{2}d^{2}+a^{2}b^{2}cd+a^{2}b^{2}cd \\ &= a^{2}b^{2}(c+d)^{2} \\ &\leqslant \frac{1}{3^{6}}(a+b+c+d)^{6} \\ &=1\end{aligned}\]
Bài toán được chứng minh. $\square$
$(b)$ Giả sử $a \geqslant b \geqslant c \geqslant d$, BĐT cần chứng minh tương đương với
$a^{3}b^{3}c^{3}+a^{3}b^{3}d^{3}+a^{3}c^{3}d^{3}+b^{3}c^{3}d^{3} \leqslant 1$
Ta đưa BĐT về dạng đồng bậc
$a^{3}b^{3}c^{3}+a^{3}b^{3}d^{3}+a^{3}c^{3}d^{3}+b^{3}c^{3}d^{3} \leqslant \frac{1}{3^9}(a+b+c)^9$
Theo BĐT $AM - GM$ ta có
$ \frac{1}{3^9}(a+b+c+d)^9 \geqslant \frac{1}{3^9}\left(3\sqrt[3]{ab(c+d)} \right)^9 = a^3b^3(c+d)^3 $
Do đó ta chỉ cần chứng minh $a^{3}b^{3}c^{3}+a^{3}b^{3}d^{3}+a^{3}c^{3}d^{3}+b^{3}c^{3}d^{3} \leqslant a^3b^3(c+d)^3$ $\Leftrightarrow c^3d^3(a^3+b^3) \leqslant 3a^3b^3cd(c+d)$ $\Leftrightarrow c^2d^2(a^3+b^3) \leqslant 3a^3b^3(c+d)$
BĐT này luôn đúng do $a \geqslant b \geqslant c \geqslant d$
Bài toán được chứng minh. $\square$$
a) $\frac{1}{a^2}+\frac{1}{b^2}+\frac{1}{c^2}+\frac{1}{d^2}\le\frac{1}{a^2b^2c^2d^2},$
b) $\frac{1}{a^3}+\frac{1}{b^3}+\frac{1}{c^3}+\frac{1}{d^3}\le\frac{1}{a^3b^3c^3d^3}.$
Lời giải
$(a)$ Giả sử $a \geqslant b \geqslant c \geqslant d$, BĐT cần chứng minh tương đương với
$a^{2}b^{2}c^{2}+a^{2}b^{2}d^{2}+a^{2}c^{2}d^{2}+b^{2}c^{2}d^{2} \leqslant 1$
Ta đưa BĐT về dạng đồng bậc
$a^{2}b^{2}c^{2}+a^{2}b^{2}d^{2}+a^{2}c^{2}d^{2}+b^{2}c^{2}d^{2} \leqslant \frac{1}{3^6}(a+b+c)^6$
Do $a \geqslant b \geqslant c \geqslant d$ nên ta có
\[\begin{aligned} a^{2}b^{2}c^{2}+a^{2}b^{2}d^{2}+a^{2}c^{2}d^{2}+b^{2}c^{2}d^{2} &\leqslant a^{2}b^{2}c^{2}+a^{2}b^{2}d^{2}+a^{2}b^{2}cd+a^{2}b^{2}cd \\ &= a^{2}b^{2}(c+d)^{2} \\ &\leqslant \frac{1}{3^{6}}(a+b+c+d)^{6} \\ &=1\end{aligned}\]
Bài toán được chứng minh. $\square$
$(b)$ Giả sử $a \geqslant b \geqslant c \geqslant d$, BĐT cần chứng minh tương đương với
$a^{3}b^{3}c^{3}+a^{3}b^{3}d^{3}+a^{3}c^{3}d^{3}+b^{3}c^{3}d^{3} \leqslant 1$
Ta đưa BĐT về dạng đồng bậc
$a^{3}b^{3}c^{3}+a^{3}b^{3}d^{3}+a^{3}c^{3}d^{3}+b^{3}c^{3}d^{3} \leqslant \frac{1}{3^9}(a+b+c)^9$
Theo BĐT $AM - GM$ ta có
$ \frac{1}{3^9}(a+b+c+d)^9 \geqslant \frac{1}{3^9}\left(3\sqrt[3]{ab(c+d)} \right)^9 = a^3b^3(c+d)^3 $
Do đó ta chỉ cần chứng minh $a^{3}b^{3}c^{3}+a^{3}b^{3}d^{3}+a^{3}c^{3}d^{3}+b^{3}c^{3}d^{3} \leqslant a^3b^3(c+d)^3$ $\Leftrightarrow c^3d^3(a^3+b^3) \leqslant 3a^3b^3cd(c+d)$ $\Leftrightarrow c^2d^2(a^3+b^3) \leqslant 3a^3b^3(c+d)$
BĐT này luôn đúng do $a \geqslant b \geqslant c \geqslant d$
Bài toán được chứng minh. $\square$$
Bất đẳng thức với số thực
Đề bài: Cho ba số thực a,b,c thỏa mãn điều kiện a2+b2+c2=4+abc. Chứng minh rằng
$(a^2+2)(b^2+2)(c^2+2)\ge 9+6(ab+bc+ca).$
Áp dụng bất đẳng thức AM-GM ta có $a^2+b^2+c^2=4+abc\geq 3\sqrt[3]{a^2b^2c^2}$
Đặt $\sqrt[3]{abc}=t\Rightarrow t^3+4\geq 3t^2\Leftrightarrow (t+1)(t-2)^2\geq 0\Leftrightarrow t\geq -1\Rightarrow abc\geq -1\Rightarrow a^2+b^2+c^2\geq 3$.
Ta sẽ chứng minh $3(a+b+c)^2\leq (a^2+2)(b^2+2)(c^2+2)$.
Ta có $(a+b+c)^2=(a.1+\sqrt{2}.\frac{b+c}{\sqrt{2}})^2\leq (a^2+2)(1+\frac{(b+c)^2}{2})$.
Do đó ta chỉ cần chứng minh $(b^2+2)(c^2+2)\geq 3(1+\frac{(b+c)^2}{2})\Leftrightarrow (bc-1)^2+\frac{(b-c)^2}{2}\geq 0$ ( luôn đúng)
Vậy $(a^2+2)(b^2+2)(c^2+2)\geq 3(a+b+c)^2=3(a^2+b^2+c^2)+6(ab+bc+ca)\geq 9+6(ab+bc+ca)$ ( do $a^2+b^2+c^2\geq 3$).
Vậy ta có đpcm.
$(a^2+2)(b^2+2)(c^2+2)\ge 9+6(ab+bc+ca).$
Lời giải:
Áp dụng bất đẳng thức AM-GM ta có $a^2+b^2+c^2=4+abc\geq 3\sqrt[3]{a^2b^2c^2}$
Đặt $\sqrt[3]{abc}=t\Rightarrow t^3+4\geq 3t^2\Leftrightarrow (t+1)(t-2)^2\geq 0\Leftrightarrow t\geq -1\Rightarrow abc\geq -1\Rightarrow a^2+b^2+c^2\geq 3$.
Ta sẽ chứng minh $3(a+b+c)^2\leq (a^2+2)(b^2+2)(c^2+2)$.
Ta có $(a+b+c)^2=(a.1+\sqrt{2}.\frac{b+c}{\sqrt{2}})^2\leq (a^2+2)(1+\frac{(b+c)^2}{2})$.
Do đó ta chỉ cần chứng minh $(b^2+2)(c^2+2)\geq 3(1+\frac{(b+c)^2}{2})\Leftrightarrow (bc-1)^2+\frac{(b-c)^2}{2}\geq 0$ ( luôn đúng)
Vậy $(a^2+2)(b^2+2)(c^2+2)\geq 3(a+b+c)^2=3(a^2+b^2+c^2)+6(ab+bc+ca)\geq 9+6(ab+bc+ca)$ ( do $a^2+b^2+c^2\geq 3$).
Vậy ta có đpcm.
Nhận xét: Ở đây ta đã sử dụng bổ đề rất quen thuộc đó là: $3(a+b+c)^2\leq (a^2+2)(b^2+2)(c^2+2)$.
Phương pháp uct trong bất đẳng thức
Bài 1: (Iran TST 2016): Cho bốn số thực dương
a,b,c,d thỏa mãn điều kiện $\frac{1}{a+1}+\frac{1}{b+1}+\frac{1}{c+1}+\frac{1}{d+1}=2.$
ta sẽ cm $\sqrt{\frac{a^{2}+1}{2}}-3\sqrt{a}\geq \frac{4}{a+1}-4$
$\Leftrightarrow \frac{(a+1)(a-1)^{2}}{2(\sqrt{\frac{a^{2}+1}{2}}+\sqrt{a})}-2\sqrt{a}(\sqrt{a}-1)^{2}\geq 0$
do $\sqrt{\frac{a^{2}+1}{2}}+\sqrt{a}\leq a+1$
nên ta cần cm $(\sqrt{a}-1)^{2}(\frac{(\sqrt{a}+1)^{2}}{2}-2\sqrt{a})\geq 0$(luôn đúng)
thiết lập các bất đẳng thức tương tự ta có đpcm
Bài 2: (Austrian 2016). Cho ba số thực
Sử dụng bất đẳng thức $a^2+a-a^3-1\leq 0\Leftrightarrow (a+1)(a-1)^2\geq 0$
Xây dựng các bất đẳng thức tượng tự rồi cộng lại ta có đpcm.
Đẳng thức xảy ra khi và chỉ khi $(a,b,c)=(1,1,-1)$ và các hoán vị.
a,b,c,d thỏa mãn điều kiện $\frac{1}{a+1}+\frac{1}{b+1}+\frac{1}{c+1}+\frac{1}{d+1}=2.$
Chứng minh: $\sqrt{\frac{a^2+1}{2}} +\sqrt{\frac{b^2+1}{2}}+\sqrt{\frac{c^2+1}
{2}}+\sqrt{\frac{d^2+1}{2}} +8 \geqslant 3(\sqrt{a}+\sqrt{b}+\sqrt{c}+\sqrt{d}).$
{2}}+\sqrt{\frac{d^2+1}{2}} +8 \geqslant 3(\sqrt{a}+\sqrt{b}+\sqrt{c}+\sqrt{d}).$
Lời giải:
Ta thấy có thể tách a,b ,c không ràng buộc với nhau và có thể đánh giá từng vế, nên ta cần tìm x và y sao cho:
$\sqrt{\frac{a^2+1}{2}} - 3\sqrt{a} \ge \frac{x}{a+1}+y$ (1)
Tương tự với b,c ta được: $2x+4y=-8$
Ngoài ra dấu bằng của $(1)$ xảy ra khi $a=b=c=d=1$ thay vào trên ta lại có hệ:
$f'(a)=0$ với $f(a)=\sqrt{\frac{a^2+1}{2}} - 3\sqrt{a} -\frac{x}{a+1}$
Giải được $x=-y=4$
ta sẽ cm $\sqrt{\frac{a^{2}+1}{2}}-3\sqrt{a}\geq \frac{4}{a+1}-4$
$\Leftrightarrow \frac{(a+1)(a-1)^{2}}{2(\sqrt{\frac{a^{2}+1}{2}}+\sqrt{a})}-2\sqrt{a}(\sqrt{a}-1)^{2}\geq 0$
do $\sqrt{\frac{a^{2}+1}{2}}+\sqrt{a}\leq a+1$
nên ta cần cm $(\sqrt{a}-1)^{2}(\frac{(\sqrt{a}+1)^{2}}{2}-2\sqrt{a})\geq 0$(luôn đúng)
thiết lập các bất đẳng thức tương tự ta có đpcm
Bài 2: (Austrian 2016). Cho ba số thực
a,b,c⩾−1 thỏa mãn điều kiện a3+b3+c3=1. Chứng minh rằnga+b+c+a2+b2+c2⩽4.
Đẳng thức xảy ra khi nào ?
Lời giải:
Sử dụng bất đẳng thức $a^2+a-a^3-1\leq 0\Leftrightarrow (a+1)(a-1)^2\geq 0$
Xây dựng các bất đẳng thức tượng tự rồi cộng lại ta có đpcm.
Đẳng thức xảy ra khi và chỉ khi $(a,b,c)=(1,1,-1)$ và các hoán vị.
Dùng cô si ngược dấu để chứng minh bất đẳng thức
Đề:Với
a,b,c là ba số thực dương thỏa mãn a+b+c=3. Chứng minh rằng:
Áp dụng AM-GM ta có:
$a-\frac{a^{2}}{a+b^{2}}=\frac{ab^{2}}{a+b^{2}}\leq \frac{ab^{2}}{2b\sqrt{a}}=\frac{b\sqrt{a}}{2}$
Tương tự cộng lại ta được:
$\sum \frac{a}{a+b^{2}}\geq 3-\frac{1}{2}(b\sqrt{a}+c\sqrt{b}+a\sqrt{c})\geq 3-\frac{1}{2}.\sqrt{(a+b+c)(ab+bc+ca)}\geq 3-\frac{1}{2}.\sqrt{(a+b+c).\frac{(a+b+c)^{2}}{3}}=3-\frac{3}{2}=\frac{3}{2}$
Dấu = xảy ra$\Leftrightarrow a=b=c=1$$
a,b,c là ba số thực dương thỏa mãn a+b+c=3. Chứng minh rằng:
$\frac{a^2}{a+b^2}+\frac{b^2}{b+c^2}+\frac{c^2}{c+a^2} \geqslant \frac{3}{2}$
Lời giải
Lời giải
Áp dụng AM-GM ta có:
$a-\frac{a^{2}}{a+b^{2}}=\frac{ab^{2}}{a+b^{2}}\leq \frac{ab^{2}}{2b\sqrt{a}}=\frac{b\sqrt{a}}{2}$
Tương tự cộng lại ta được:
$\sum \frac{a}{a+b^{2}}\geq 3-\frac{1}{2}(b\sqrt{a}+c\sqrt{b}+a\sqrt{c})\geq 3-\frac{1}{2}.\sqrt{(a+b+c)(ab+bc+ca)}\geq 3-\frac{1}{2}.\sqrt{(a+b+c).\frac{(a+b+c)^{2}}{3}}=3-\frac{3}{2}=\frac{3}{2}$
Dấu = xảy ra$\Leftrightarrow a=b=c=1$$
Đăng ký:
Bài đăng (Atom)
Bất đẳng thức tuyển sinh lớp 10 chọn lọc
Trong bài viết này, tác giả giới thiệu một số bài BĐT nhẹ nhàng nhưng ý tưởng tương đối mới, mức độ phù hợp với đề thi tuyển sinh vào lớp...
-
I) Hàm phần nguyên: 1) Định nghĩa Phần nguyên của một số thực x là số nguyên lớn nhất không vượt quá x. Kí hiệu là [x]. 2) Tính chất...
-
Trong thế giới bất đẳng thức , ngoài những bất đẳng thức kinh điển và được áp dụng rất nhiều như bất đẳng thức AM – GM, bất đẳng thức Cauc...
-
Định nghĩa 1: Một số nguyên a được gọi là thặng dư bình phương mod n nếu tồn tại số nguyên x sao cho $x^2 \equiv a (mod n)$ Ta cũng có th...